6
$\begingroup$

In this question the OP asks whether the sum $$ f(q, \alpha) = \sum _{k=1}^{\infty } \frac{q^k \left(q^k-1\right)^\alpha}{(q;q)_k} $$ is ever zero. An experiment with Mathematica indicates, to any reasonable precision that $$f(2, 1) = \sum _{k=1}^{\infty } \frac{2^k }{(2;2)_{k-1}} = 0,$$ but I, for one, can't actually prove it. Is it true?

$\endgroup$
0

2 Answers 2

6
$\begingroup$

$$1/(2-1)\dots (2^{k-1}-1)+1/(2-1)\dots (2^k-1)=2^k/(2-1)\dots (2^k-1),$$ thus alternating sum of expression on the right is telescopical.

$\endgroup$
5
$\begingroup$

We can use the Euler identity $\sum_{k=0}^\infty \frac{x^k}{q^{k(k-1)/2}(1/q;1/q)_k}=\prod_{j=0}^\infty(1+q^{-j}x)$ for $q>1$. Taking $x=-1$, we obtain the series $f(q,1)$ in the original post, and by the infinite product representation it equals 0.

$\endgroup$
5
  • $\begingroup$ Can you give a precise reference for this Euler identity? $\endgroup$
    – GH from MO
    Sep 4, 2015 at 18:53
  • 1
    $\begingroup$ @GHfromMO, A quick way to prove it is to notice that the coefficient of $x^kq^{-n}$ on both sides is the number of partitions of $n$ into $k$ distinct parts. $\endgroup$ Sep 4, 2015 at 18:58
  • 1
    $\begingroup$ @GHfromMO One of the possible references is: G. E. Andrews, R. Askey, R. Roy, Special Functions, Cambridge Univ. Press, Cambridge 1999, Corollary 10.2.2 (b) on page 490. It is also presented in the Books by Kac&Cheung, and Gasper&Rahman, but I don't have the exact pages at the moment (Can provşde later on) $\endgroup$
    – Deepti
    Sep 4, 2015 at 19:10
  • $\begingroup$ @Deepti: Thank you! $\endgroup$
    – GH from MO
    Sep 4, 2015 at 19:29
  • $\begingroup$ I think we need to plug in $x=-1$ instead of $x=-2$, and this works for any $q>1$. This is because your sum on the left hand side equals $\sum_{k=0}^\infty\frac{(-qx)^k}{(q;q)_k}$. $\endgroup$
    – GH from MO
    Sep 4, 2015 at 21:05

Your Answer

By clicking “Post Your Answer”, you agree to our terms of service and acknowledge you have read our privacy policy.

Not the answer you're looking for? Browse other questions tagged or ask your own question.